VMMC VMMC Medical Solved Paper-2014

  • question_answer
    Effective capacitance between A and B in the figure shown is \[({{C}_{1}}={{C}_{2}}=20\mu F,{{C}_{3}}={{C}_{4}}=10\mu F)\]

    A) \[10\mu F\]                       

    B) \[15\mu F\]

    C) \[20\mu F\]                       

    D) \[25\mu F\]

    Correct Answer: B

    Solution :

    Here, \[{{C}_{1}}\]and \[{{C}_{2}}\]are in series. Hence, their effective capacitance C" is given by \[\frac{1}{C'}=\frac{1}{{{C}_{1}}}=\frac{1}{{{C}_{2}}}\] \[\frac{1}{C'}=\frac{1}{20}+\frac{1}{20}\]\[\Rightarrow \]\[C'=10\mu F\] Similarly,  \[\frac{1}{C''}=\frac{1}{{{C}_{3}}}+\frac{1}{{{C}_{4}}}\] \[\frac{1}{C''}=\frac{1}{10}+\frac{1}{10}\]\[\Rightarrow \]\[C''=5\mu F\] Now \[C'\]and \[C''\]are in parallel. Hence, resultant capacitance C will be \[C=C'+C''\] \[=10+5=15\mu F\]


You need to login to perform this action.
You will be redirected in 3 sec spinner